Prove uniform convergence of $f_{n} (x) = sqrt{frac{1}{n^2} +x^2}$ to $f(x) = |x|$












1












$begingroup$



For each $n ∈ Bbb N$, let $f_{n} (x) = sqrt{frac{1}{n^2} +x^2}. $ Show $(f_{n})^{infty}_{n=0}$ converges uniformly on $Bbb R$ to $f(x) = |x|$




I'm trying to find the $N$ such that for all $x in Bbb R$ and $n > N$, $$Bigg|sqrt{frac{1}{n^2} +x^2} - |x|Bigg| < epsilon$$ but I'm struggling with proving convergence for all real $x$. I'v seen that for $x∈ [0,1]$, for $n$ greater than or equal to $1$, $$Bigg|sqrt{frac{1}{n^2} +x^2} - |x|Bigg|leq frac{1}{sqrt{n}} $$ but I'm unsure of how to generalize. Any help is appreciated!










share|cite|improve this question











$endgroup$

















    1












    $begingroup$



    For each $n ∈ Bbb N$, let $f_{n} (x) = sqrt{frac{1}{n^2} +x^2}. $ Show $(f_{n})^{infty}_{n=0}$ converges uniformly on $Bbb R$ to $f(x) = |x|$




    I'm trying to find the $N$ such that for all $x in Bbb R$ and $n > N$, $$Bigg|sqrt{frac{1}{n^2} +x^2} - |x|Bigg| < epsilon$$ but I'm struggling with proving convergence for all real $x$. I'v seen that for $x∈ [0,1]$, for $n$ greater than or equal to $1$, $$Bigg|sqrt{frac{1}{n^2} +x^2} - |x|Bigg|leq frac{1}{sqrt{n}} $$ but I'm unsure of how to generalize. Any help is appreciated!










    share|cite|improve this question











    $endgroup$















      1












      1








      1





      $begingroup$



      For each $n ∈ Bbb N$, let $f_{n} (x) = sqrt{frac{1}{n^2} +x^2}. $ Show $(f_{n})^{infty}_{n=0}$ converges uniformly on $Bbb R$ to $f(x) = |x|$




      I'm trying to find the $N$ such that for all $x in Bbb R$ and $n > N$, $$Bigg|sqrt{frac{1}{n^2} +x^2} - |x|Bigg| < epsilon$$ but I'm struggling with proving convergence for all real $x$. I'v seen that for $x∈ [0,1]$, for $n$ greater than or equal to $1$, $$Bigg|sqrt{frac{1}{n^2} +x^2} - |x|Bigg|leq frac{1}{sqrt{n}} $$ but I'm unsure of how to generalize. Any help is appreciated!










      share|cite|improve this question











      $endgroup$





      For each $n ∈ Bbb N$, let $f_{n} (x) = sqrt{frac{1}{n^2} +x^2}. $ Show $(f_{n})^{infty}_{n=0}$ converges uniformly on $Bbb R$ to $f(x) = |x|$




      I'm trying to find the $N$ such that for all $x in Bbb R$ and $n > N$, $$Bigg|sqrt{frac{1}{n^2} +x^2} - |x|Bigg| < epsilon$$ but I'm struggling with proving convergence for all real $x$. I'v seen that for $x∈ [0,1]$, for $n$ greater than or equal to $1$, $$Bigg|sqrt{frac{1}{n^2} +x^2} - |x|Bigg|leq frac{1}{sqrt{n}} $$ but I'm unsure of how to generalize. Any help is appreciated!







      real-analysis sequences-and-series uniform-convergence






      share|cite|improve this question















      share|cite|improve this question













      share|cite|improve this question




      share|cite|improve this question








      edited Dec 5 '18 at 9:36









      Chinnapparaj R

      5,4331928




      5,4331928










      asked Dec 5 '18 at 9:28









      user613048user613048

      322




      322






















          4 Answers
          4






          active

          oldest

          votes


















          1












          $begingroup$

          Note that$$sqrt{x^2+frac1{n^2}}-lvert xrvert=frac{frac1{n^2}}{sqrt{x^2+frac1{n^2}}+lvert xrvert}.$$Since the numerator is constant and the denominator is positive and increases to $infty$ when $lvert xrverttoinfty$, the maximum of $sqrt{x^2+frac1{n^2}}-lvert xrvert$ is attained when $x=0$. And that maximum is $frac1n$. So$$(forall xinmathbb{R}):leftlvertsqrt{x^2+frac1{n^2}}-lvert xrvertrightrvertleqslantfrac1n,$$from which the uniform convergence follows.






          share|cite|improve this answer









          $endgroup$





















            1












            $begingroup$

            If you use $sqrt a -sqrt b=frac {a-b} {sqrt a +sqrt b}$ you will get the result easily.






            share|cite|improve this answer









            $endgroup$





















              1












              $begingroup$

              Hint: We have
              $$|f_n(x) - |x|| = frac{1}{n^2} frac{1}{|x|+sqrt{1/n^2 +x^2}} le frac{1}{n}.$$






              share|cite|improve this answer









              $endgroup$





















                1












                $begingroup$

                Hint: $$left(vert x vert +frac{1}{n}right)^2=x^2+frac{1}{n^2}+2vert x vert frac{1}{n}=f_n^2+2vert x vert frac{1}{n} geq f_n^2 geq 0$$ so $$0 leq f_n le vert x vert+frac{1}{n}$$ and so $$0 leq f_n - vert x vert leq frac{1}{n} rightarrow 0$$





                Moral:



                Uniform limit of sequence of continuously differentiable function need not be continuously differentiable!






                share|cite|improve this answer











                $endgroup$













                  Your Answer





                  StackExchange.ifUsing("editor", function () {
                  return StackExchange.using("mathjaxEditing", function () {
                  StackExchange.MarkdownEditor.creationCallbacks.add(function (editor, postfix) {
                  StackExchange.mathjaxEditing.prepareWmdForMathJax(editor, postfix, [["$", "$"], ["\\(","\\)"]]);
                  });
                  });
                  }, "mathjax-editing");

                  StackExchange.ready(function() {
                  var channelOptions = {
                  tags: "".split(" "),
                  id: "69"
                  };
                  initTagRenderer("".split(" "), "".split(" "), channelOptions);

                  StackExchange.using("externalEditor", function() {
                  // Have to fire editor after snippets, if snippets enabled
                  if (StackExchange.settings.snippets.snippetsEnabled) {
                  StackExchange.using("snippets", function() {
                  createEditor();
                  });
                  }
                  else {
                  createEditor();
                  }
                  });

                  function createEditor() {
                  StackExchange.prepareEditor({
                  heartbeatType: 'answer',
                  autoActivateHeartbeat: false,
                  convertImagesToLinks: true,
                  noModals: true,
                  showLowRepImageUploadWarning: true,
                  reputationToPostImages: 10,
                  bindNavPrevention: true,
                  postfix: "",
                  imageUploader: {
                  brandingHtml: "Powered by u003ca class="icon-imgur-white" href="https://imgur.com/"u003eu003c/au003e",
                  contentPolicyHtml: "User contributions licensed under u003ca href="https://creativecommons.org/licenses/by-sa/3.0/"u003ecc by-sa 3.0 with attribution requiredu003c/au003e u003ca href="https://stackoverflow.com/legal/content-policy"u003e(content policy)u003c/au003e",
                  allowUrls: true
                  },
                  noCode: true, onDemand: true,
                  discardSelector: ".discard-answer"
                  ,immediatelyShowMarkdownHelp:true
                  });


                  }
                  });














                  draft saved

                  draft discarded


















                  StackExchange.ready(
                  function () {
                  StackExchange.openid.initPostLogin('.new-post-login', 'https%3a%2f%2fmath.stackexchange.com%2fquestions%2f3026863%2fprove-uniform-convergence-of-f-n-x-sqrt-frac1n2-x2-to-fx%23new-answer', 'question_page');
                  }
                  );

                  Post as a guest















                  Required, but never shown

























                  4 Answers
                  4






                  active

                  oldest

                  votes








                  4 Answers
                  4






                  active

                  oldest

                  votes









                  active

                  oldest

                  votes






                  active

                  oldest

                  votes









                  1












                  $begingroup$

                  Note that$$sqrt{x^2+frac1{n^2}}-lvert xrvert=frac{frac1{n^2}}{sqrt{x^2+frac1{n^2}}+lvert xrvert}.$$Since the numerator is constant and the denominator is positive and increases to $infty$ when $lvert xrverttoinfty$, the maximum of $sqrt{x^2+frac1{n^2}}-lvert xrvert$ is attained when $x=0$. And that maximum is $frac1n$. So$$(forall xinmathbb{R}):leftlvertsqrt{x^2+frac1{n^2}}-lvert xrvertrightrvertleqslantfrac1n,$$from which the uniform convergence follows.






                  share|cite|improve this answer









                  $endgroup$


















                    1












                    $begingroup$

                    Note that$$sqrt{x^2+frac1{n^2}}-lvert xrvert=frac{frac1{n^2}}{sqrt{x^2+frac1{n^2}}+lvert xrvert}.$$Since the numerator is constant and the denominator is positive and increases to $infty$ when $lvert xrverttoinfty$, the maximum of $sqrt{x^2+frac1{n^2}}-lvert xrvert$ is attained when $x=0$. And that maximum is $frac1n$. So$$(forall xinmathbb{R}):leftlvertsqrt{x^2+frac1{n^2}}-lvert xrvertrightrvertleqslantfrac1n,$$from which the uniform convergence follows.






                    share|cite|improve this answer









                    $endgroup$
















                      1












                      1








                      1





                      $begingroup$

                      Note that$$sqrt{x^2+frac1{n^2}}-lvert xrvert=frac{frac1{n^2}}{sqrt{x^2+frac1{n^2}}+lvert xrvert}.$$Since the numerator is constant and the denominator is positive and increases to $infty$ when $lvert xrverttoinfty$, the maximum of $sqrt{x^2+frac1{n^2}}-lvert xrvert$ is attained when $x=0$. And that maximum is $frac1n$. So$$(forall xinmathbb{R}):leftlvertsqrt{x^2+frac1{n^2}}-lvert xrvertrightrvertleqslantfrac1n,$$from which the uniform convergence follows.






                      share|cite|improve this answer









                      $endgroup$



                      Note that$$sqrt{x^2+frac1{n^2}}-lvert xrvert=frac{frac1{n^2}}{sqrt{x^2+frac1{n^2}}+lvert xrvert}.$$Since the numerator is constant and the denominator is positive and increases to $infty$ when $lvert xrverttoinfty$, the maximum of $sqrt{x^2+frac1{n^2}}-lvert xrvert$ is attained when $x=0$. And that maximum is $frac1n$. So$$(forall xinmathbb{R}):leftlvertsqrt{x^2+frac1{n^2}}-lvert xrvertrightrvertleqslantfrac1n,$$from which the uniform convergence follows.







                      share|cite|improve this answer












                      share|cite|improve this answer



                      share|cite|improve this answer










                      answered Dec 5 '18 at 9:35









                      José Carlos SantosJosé Carlos Santos

                      159k22126229




                      159k22126229























                          1












                          $begingroup$

                          If you use $sqrt a -sqrt b=frac {a-b} {sqrt a +sqrt b}$ you will get the result easily.






                          share|cite|improve this answer









                          $endgroup$


















                            1












                            $begingroup$

                            If you use $sqrt a -sqrt b=frac {a-b} {sqrt a +sqrt b}$ you will get the result easily.






                            share|cite|improve this answer









                            $endgroup$
















                              1












                              1








                              1





                              $begingroup$

                              If you use $sqrt a -sqrt b=frac {a-b} {sqrt a +sqrt b}$ you will get the result easily.






                              share|cite|improve this answer









                              $endgroup$



                              If you use $sqrt a -sqrt b=frac {a-b} {sqrt a +sqrt b}$ you will get the result easily.







                              share|cite|improve this answer












                              share|cite|improve this answer



                              share|cite|improve this answer










                              answered Dec 5 '18 at 9:31









                              Kavi Rama MurthyKavi Rama Murthy

                              57.7k42160




                              57.7k42160























                                  1












                                  $begingroup$

                                  Hint: We have
                                  $$|f_n(x) - |x|| = frac{1}{n^2} frac{1}{|x|+sqrt{1/n^2 +x^2}} le frac{1}{n}.$$






                                  share|cite|improve this answer









                                  $endgroup$


















                                    1












                                    $begingroup$

                                    Hint: We have
                                    $$|f_n(x) - |x|| = frac{1}{n^2} frac{1}{|x|+sqrt{1/n^2 +x^2}} le frac{1}{n}.$$






                                    share|cite|improve this answer









                                    $endgroup$
















                                      1












                                      1








                                      1





                                      $begingroup$

                                      Hint: We have
                                      $$|f_n(x) - |x|| = frac{1}{n^2} frac{1}{|x|+sqrt{1/n^2 +x^2}} le frac{1}{n}.$$






                                      share|cite|improve this answer









                                      $endgroup$



                                      Hint: We have
                                      $$|f_n(x) - |x|| = frac{1}{n^2} frac{1}{|x|+sqrt{1/n^2 +x^2}} le frac{1}{n}.$$







                                      share|cite|improve this answer












                                      share|cite|improve this answer



                                      share|cite|improve this answer










                                      answered Dec 5 '18 at 9:32









                                      p4schp4sch

                                      5,245217




                                      5,245217























                                          1












                                          $begingroup$

                                          Hint: $$left(vert x vert +frac{1}{n}right)^2=x^2+frac{1}{n^2}+2vert x vert frac{1}{n}=f_n^2+2vert x vert frac{1}{n} geq f_n^2 geq 0$$ so $$0 leq f_n le vert x vert+frac{1}{n}$$ and so $$0 leq f_n - vert x vert leq frac{1}{n} rightarrow 0$$





                                          Moral:



                                          Uniform limit of sequence of continuously differentiable function need not be continuously differentiable!






                                          share|cite|improve this answer











                                          $endgroup$


















                                            1












                                            $begingroup$

                                            Hint: $$left(vert x vert +frac{1}{n}right)^2=x^2+frac{1}{n^2}+2vert x vert frac{1}{n}=f_n^2+2vert x vert frac{1}{n} geq f_n^2 geq 0$$ so $$0 leq f_n le vert x vert+frac{1}{n}$$ and so $$0 leq f_n - vert x vert leq frac{1}{n} rightarrow 0$$





                                            Moral:



                                            Uniform limit of sequence of continuously differentiable function need not be continuously differentiable!






                                            share|cite|improve this answer











                                            $endgroup$
















                                              1












                                              1








                                              1





                                              $begingroup$

                                              Hint: $$left(vert x vert +frac{1}{n}right)^2=x^2+frac{1}{n^2}+2vert x vert frac{1}{n}=f_n^2+2vert x vert frac{1}{n} geq f_n^2 geq 0$$ so $$0 leq f_n le vert x vert+frac{1}{n}$$ and so $$0 leq f_n - vert x vert leq frac{1}{n} rightarrow 0$$





                                              Moral:



                                              Uniform limit of sequence of continuously differentiable function need not be continuously differentiable!






                                              share|cite|improve this answer











                                              $endgroup$



                                              Hint: $$left(vert x vert +frac{1}{n}right)^2=x^2+frac{1}{n^2}+2vert x vert frac{1}{n}=f_n^2+2vert x vert frac{1}{n} geq f_n^2 geq 0$$ so $$0 leq f_n le vert x vert+frac{1}{n}$$ and so $$0 leq f_n - vert x vert leq frac{1}{n} rightarrow 0$$





                                              Moral:



                                              Uniform limit of sequence of continuously differentiable function need not be continuously differentiable!







                                              share|cite|improve this answer














                                              share|cite|improve this answer



                                              share|cite|improve this answer








                                              edited Dec 5 '18 at 9:42

























                                              answered Dec 5 '18 at 9:33









                                              Chinnapparaj RChinnapparaj R

                                              5,4331928




                                              5,4331928






























                                                  draft saved

                                                  draft discarded




















































                                                  Thanks for contributing an answer to Mathematics Stack Exchange!


                                                  • Please be sure to answer the question. Provide details and share your research!

                                                  But avoid



                                                  • Asking for help, clarification, or responding to other answers.

                                                  • Making statements based on opinion; back them up with references or personal experience.


                                                  Use MathJax to format equations. MathJax reference.


                                                  To learn more, see our tips on writing great answers.




                                                  draft saved


                                                  draft discarded














                                                  StackExchange.ready(
                                                  function () {
                                                  StackExchange.openid.initPostLogin('.new-post-login', 'https%3a%2f%2fmath.stackexchange.com%2fquestions%2f3026863%2fprove-uniform-convergence-of-f-n-x-sqrt-frac1n2-x2-to-fx%23new-answer', 'question_page');
                                                  }
                                                  );

                                                  Post as a guest















                                                  Required, but never shown





















































                                                  Required, but never shown














                                                  Required, but never shown












                                                  Required, but never shown







                                                  Required, but never shown

































                                                  Required, but never shown














                                                  Required, but never shown












                                                  Required, but never shown







                                                  Required, but never shown







                                                  Popular posts from this blog

                                                  Plaza Victoria

                                                  In PowerPoint, is there a keyboard shortcut for bulleted / numbered list?

                                                  How to put 3 figures in Latex with 2 figures side by side and 1 below these side by side images but in...